What is 7.96 repeating as a mixed number in simplest form

Answers

Answer 1

Answer:

Ion even know jus need points

Step-by-step explanation:


Related Questions

What is the simplified form of 3a^2+6a+2b^2.

It would be greatly appreciated if anyone could solve this

Answers

Answer:

3 a (a + 2) + 2 b^2 or a (3 a + 6) + 2 b^2 or 3 (a + 1)^2 + 2 b^2 - 3

Step-by-step explanation:

this is due tm please help​

Answers

Answer:

25. No slope

Step-by-step explanation:

25. Y2 - Y1 / X2 - X1

= -8 - (-3) / 9 - 9

= -5 / 0

= no slope; straight line

Use this formula (Y2 - Y1 / X2 - X1) for each ques.

How could you prove that a number is divisible by 16? Use the space below to test your hypothesis using the numbers 144 & 256. Explain your new rule.

Answers

Answer:

We factor the number 16 into prime numbers to get their factors.

We do the same with any given number. Only if it has the same factor as 16 that numberwill be divisible by 16

Both numbers proposed (144&256) are divisible by 16

Step-by-step explanation:

To prove it we need to factor the number 16 to get their expression in prime numbers:

16     2

 8    2

 4    2

 2    2

 1

16 is equal to [tex]2^{4}[/tex]

We have to factor a number and if their factor include [tex]2^{4}[/tex] or a higher power

then, they are divisible by 16

144        2

 72       2

 36       2

 18        2

   9       3

   3       3

   1

144 is queal to:  [tex]2^4 . 3^2[/tex] as it does have [tex]2^4[/tex] It is divisible by 16

256         2

128          2

  64        2

  32        2

   16        2

    8        2

    4        2

    2        2

    1

256 is equal to [tex]2^8[/tex] This contains [tex]2^4[/tex] as [tex]2^8 = 2^4 . 2^4[/tex]

therefore it is divisible by 16

Write a compound inequality that represents the sentence below.
A quantity x is either less than 2 or greater than 16.

Answers

Answer:

2<x>16

Step-by-step explanation:

Help me do the ones marked in red.

Answers

Answer:

  2. "two thirds of the fourth power of r"

  4.  n + 14

  6.  7 +11n

  8.  (2/5)n^2

__

  7.  28

  8.  1/5

Step-by-step explanation:

2. "two thirds of the fourth power of r"

__

4. "sum" means the listed items are added. You can use any convenient variable to represent "a number." I often use n for number.

  n + 14

__

6. "7 more than" means 7 is added to whatever follows. "11 times a number" means the number is multiplied by 11.

  7 + 11n

__

8. "of" means "times". The square of a number is that number to the power of 2.

  (2/5)n^2

__

7 & 8 (evaluation). Your calculator can do this. (If not, get one that can.) Note that parentheses are needed around numerators and denominators and anything else that must be treated as a single value.

Please help and show work! Simplify each expression!




-7y - (-12y) =



15xy- (-6xy) =



-53va - 32va



Answer the expressions and show work please!



64 / 0.8 -5.6 / 7 =



Use the distributive property to simplify the expression. Show work!



-(x -3) + 6 =



Use order of operations to evaluate the expression.



Show work please!



6 + 3 (9) =



Evaluate the expressions if x = 10, y = 5, and z = 1.


x/y =



xy + z



5 (z - x)



Please answer all of these questions correctly and show work! I really need this! I'm putting 50 points and I'll choose brainliest!

Answers

Answer:

See below

Step-by-step explanation:

-7y - (-12y) =  -7y + 12y = (-7+12)y = 5y  15xy- (-6xy) =  15xy + 6xy = (15+6)xy = 21xy-53va - 32va  = (-53 -32)va = - 85va64 / 0.8 - 5.6 / 7 = 640/8 - 56/7×1/10 = 80 - 8/10 = 80 - 0.8 = 79.2-(x -3) + 6 = -x + 3 + 6 = -x + 96 + 3 (9) = 6 + 27 = 33

Evaluate the expressions if x = 10, y = 5, and z = 1.

x/y =  10/5 = 2xy + z  = 10×5 + 1 = 50 + 1 = 515 (z - x) = 5×(1 - 10) = 5×(-9) = -45

Answer:

see below

Step-by-step explanation:

⇒  -7y - (-12y) = -7y + 12y =  5y

⇒  15xy - (-6xy) = 15xy + 6xy = 21xy

⇒  -53va - 32va =    - 85av

⇒  64 / 0.8 -5.6 / 7

   = 448 - 7.48

          5*6

    = 79.2

⇒    -(x -3) + 6

   = -x + 3 + 6

   = -x + 9

⇒  6 + 3 (9)

   = 6 + 27

   = 33

Evaluate the expressions if x = 10, y = 5, and z = 1.

⇒  x/y = 10 / 5 = 2

⇒  xy + z = 10*5 + 1 = 51

⇒  5 (z - x) = 5(1 - 10) = 5 - 50 = 45

Khairul is x years old .Khairul`s uncle is 4 times as old as khairul will be in 5 years .Find the present age of Kharuls`s uncle. Plz some one answer this question will be very thankfull

Answers

Answer:

4x + 15

Step-by-step explanation:

Let the age of Khairul's uncle y.

From the question given above, the following were obtained:

Khairul's age = x years

In 5 years time:

Khairul's age = x + 5

Khairul's uncle age = y + 5

But from the question given above,

Khairul`s uncle will be 4 times as old as what khairul will be in 5 years. This can be written as follow:

y + 5 = 4(x + 5)

Clear bracket

y + 5 = 4x + 20

Subtract 5 from both side

y + 5 – 5 = 4x + 20 – 5

y = 4x + 15

Therefore, Khairul's uncle present age is 4x + 15

Answer:

y= 4x+15

Step-by-step explanation:

This problem tests our ability to express word problem using equation

Given data

Khairul age = x years

let the uncle's age be y

in five years time, Khairul will be = x+5

in five years time, the uncle will be = y+5

 

hence Khairul uncle's age  is  

y= 4(x+5)

y=4(x+5)

y=4x+20

hence the uncles current age is

y+5=4x+20

y= 4x+20-5

y= 4x+15

   

Help ASAP !!!!!!!!!!!!!!!!!!!

Answers

Answer:

im lik 999999999

percent sure its 2

Step-by-step explanation:

Answer:

divide r by s and multiply by 3   :3   :8    xD

Step-by-step explanation:

The height of a triangle is 5 more than double the base length. If the height of the triangle is 74 mm, what is the base length of the triangle? If b represents the length of the base in mm, the equation that represents this problem is

Answers

Answer:

The base length of the triangle is 34.5 mm.

34.5×2+5=74.

Step-by-step explanation:

Subtract 5 from 74 then divide 2 from 69 and you will get 34.5.

Answer:

The first answer is 74=2b+5

And the second answer is 34.5

Step-by-step explanation:

quadrilateral A can be rotated into position of quadrilateral B. Estimate the angle of rotation​

Answers

Answer:

The angle of rotation is 120°

Step-by-step explanation:

The given quadrilateral A can be taken as being upright form, however, by measurement, using a protractor application, the base of the quadrilateral is inclined at an angle of 13° to the horizontal, the positive direction of the x-axis, before rotation while the base of the B which is the image of A after rotation, is found to be at angle of 133° to the positive direction of the horizontal x-axis

Therefore, the angle of rotation = 133° - 13° = 120°

2 ABC Company has a large piece of equipment
that cost $85,600 when it was first purchased 6
years ago. The current value of the equipment is
$30,400. What is the average depreciation of the
equipment per year?
F. $ 5,800
G. $ 9,200
H. $15,200
J. $27,600
K. $42,800

Answers

Answer:5,800

Step-by-step explanation:

The average depreciation of the equipment per year is $9200.

What is depreciation?

The monetary value of an asset decreases over time due to use, wear and tear or obsolescence, this is called as depreciation.

Now it is given that,

Cost of the equipment = $85,600

current value of the equipment is given as $30,400

Duration for which the equipment was used = 6 years

Now the depreciation of the equipment is given as,

Total depreciation in the price of the equipment

                       = Cost of the equipment - current price of the equipment

So, total depreciation = $85,600 - $30,400 = $55,200

Now,

Average yearly depreciation = Total depreciation ÷ Years used

Average depreciation = 55,200/6

⇒Average depreciation = $9200

Therefore, average depreciation of the equipment per year is $9200.

More about depreciation :

https://brainly.com/question/9434632

#SPJ5

Is due today Help!!! Algebra ll

Answers

[tex]-7x-9y=-8[/tex]

[tex]5x+6y=4[/tex]

[tex]-35x-45y=-40[/tex]

[tex]35x+42y=28[/tex]

[tex]-3y=-12[/tex]

[tex]y=4[/tex]

[tex]5x+6(4)=4[/tex]

[tex]5x+24=4[/tex]

[tex]5x=-20[/tex]

[tex]x=-4[/tex]

[tex](-4,4)[/tex]

Hope this helps.

頑張って!

Find the area and perimeter of the figure below.

Answers

Answer:

Area: 6x+6

Perimiter: 4x+10

Step-by-step explanation: to find the area of something multiply the width times the height. to find the perimeter of something add each side together. use the order of operations (PEMDAS).

Area=3(2x+2)

Area=6x+6

Perimeter=3+3+(2x+2)+(2x+2)

Perimeter=6+4x+4

Perimeter=4x+10

Answer:

Area=length*width

A=3(2x+2)=6x+6 unit squar

Perimeter = 2l+2w

P=2(3)+2(2x+2)

P=6+4x+4

P=10+4x

will someone help with homework

Answers

i believe the answer is -79.31!
i got $-61.58 for her bank total

HELP ME PLS LOVE U GUYS ,<322

Answers

Answer:

384

Step-by-step explanation:

Triangle sides: 12x10/2. Then multiply by four to get all four sides.

Bottom: 12x12.

Add together, get 384.

The newest model of a phone comes with several customizable options, including 4 different sizes, 3 different storage capacities, and 6 different colors. If Gael wants to buy 1 of the 2 largest sizes of phone, how many possible combinations of options can he choose from?

Answers

13 different choices!

Only #6ac please explain.

Answers

Answer:

Step-by-step explanation:

the percentage error: P

Approximate value=a 1.41

Exact = e(1.41421356 or √2)

p=[(e-a)/e]×100

p=[√2-1.41)/√2]×100

p=0.29794385 ≅0.30 or 30 percent when rounded to the nearest hundredth

b: p=[(π-3.14)/π]×100=0.05069573≅0.05 =5 percent when rounded to the nearest hundredth. in the pi case the percentage of error is small 5 % but in the case of square roots the percentage id higher(30%). in this case it is better to round the answer at the end of calculation( math problem)

c) sometimes when rounding square roots or irrational number like pi it is useful to use decimal approximate, in the case of pi the percentage rate was 5 percent which is very low comparing to the square root which is 30 %.

GHKL is a rectangle. which of the following is a false statement?

Answers

Answer:

The false statement is ∠1 ≅ ∠2.

Evaluate this question (2/5^3)

Answers

Answer:

The answer will be 18/125 and the decimal form is 0.064

Step-by-step explanation:

3x - (2 - x) + 1
or
-5 + 4x + 4
Which is greater

Answers

Answer:

The first one is greater in value

Step-by-step explanation:

3 x 2 + x − 2 VS 4 x − 1

3x - (2 - x) + 1 is greater

oooooooooooooooooooooooooooooooooooooooooooooooooooo

Answers

Oooooooooooooooooooooooooooooooooooooooooooooooooooooooooooooooo
Ahhhhhhhhhhhhhhhhhhhhhhhh

list the numbers in order from greatest to least -1. 7, -15, -7, 2, 1

Answers

7,2,1,-1,-7,-15 that’s greatest to least

The figure shows two triangles on a coordinate grid: A coordinate grid is shown from positive 6 to negative 6 on the x-axis and from positive 6 to negative 6 on the y-axis. A triangle ABC is shown with vertex A on ordered pair negative 4, 1 vertex B on ordered pair negative 3,1, and vertex C on ordered pair negative 4, 4. Another triangle A prime B prime C prime is shown with vertex A prime on ordered pair 4, 4, vertex B prime on ordered pair 3, 4, and vertex C prime on ordered pair 4,1. What set of transformations is performed on triangle ABC to form triangle A′B′C′? (5 points) A 180-degree counterclockwise rotation about the origin, followed by a translation 5 units down A translation 5 units down followed by a 180-degree counterclockwise rotation about the origin A 270-degree counterclockwise rotation about the origin, followed by a translation 5 units to the right A translation 5 units to the right, followed by a 270-degree counterclockwise rotation about the origin

Answers

Answer:

  A translation 5 units down followed by a 180-degree counterclockwise rotation about the origin

Step-by-step explanation:

The figure is obviously rotated 180°. If the rotation is first, the image would be in the 4th quadrant, where it would stay after translation downward.

If the translation is downward first, it would be in the 3rd quadrant, moving to the first quadrant after rotation by 180°. This is the apparent sequence of transformations:

  translation down 5, rotation 180°

Answer:

translation down 5, rotation 180°

Step-by-step explanation:

Nela is filling her circular backyard swimming pool. The radius of the pool is 5 feet and the height is 6 feet. Which shows the correct substitution of these values into the formula for the volume of a cylinder?

Answers

Answer:

V=[tex]\pi[/tex]5^2*6

Step-by-step explanation:

If this makes it a little easier to read:

V = [tex]\pi[/tex] * 5^2 * 6

This is the answer when you solve it btw:

471.24

Also, who buys a swimming pool in the shape of a cylinder?

The volume of the cylinder is 471 square feet.

Given,

Nela is filling her circular backyard swimming pool.

The radius of the pool is 5 feet and the height is 6 feet.

We need to find the volume of the cylinder.

What is the volume of a cylinder?

It is given by:

V =  [tex]\pi[/tex] r² h.

Find the volume of the cylinder

V =  [tex]\pi[/tex] r² h

We have,

h = 6 feet and r = 5 feet

V = 3.14 x 5² x 6

V = 3.14 x 25 x 6

V = 471 square feet.

Thus the volume of the cylinder is 471 square feet.

Learn more about finding the volume of a cylinder here:

https://brainly.com/question/15002580

#SPJ2

Plzz look at this problem first person with correct answer will get branliest

Answers

Answer:

h,  negative,  negative

Step-by-step explanation:

Which of the folloving statements is the most precise and corred to define perpendicular lines?

Answers

Answer:C

Step-by-step explanation:

Which answer describes the type of numbers that are dense? whole numbers and integers whole numbers but not integers rational numbers and irrational numbers rational numbers but not irrational numbers

Answers

Answer -rational numbers and irrational numbers

Using number sets, it is found that the set of dense numbers is composed of rational and irrational numbers. Therefore, option C is the correct answer.

From options, we need to check which answer describes the type of numbers that are dense.

What are numbers?

Numbers are an integral part of our everyday lives, right from the number of hours we sleep at night to the number of rounds we run around the racing track and much more. In math, numbers can be even and odd numbers, prime and composite numbers, decimals, fractions, rational and irrational numbers, natural numbers, integers, real numbers, rational numbers, irrational numbers, and whole numbers.

Numbers can be classified as:

Whole numbers: All positive numbers and 0, so: {0,1,2,...}

Integer numbers: Positive or negative, not decimal so: {...,-2,-1,0,1,2,....}

Rational numbers: Integer plus decimals that can be represented by fractions, that is, they either have a pattern or have a finite number of decimal digits, for example, 0, 2, 0,45(finite number of decimal digits), 0.3333(3 repeating is the pattern), 0.32344594459 (4459 repeating is the pattern).

Irrational numbers: Decimal numbers that are not represented by patterns, that is, for example, 0.1033430290339, which can be approximated to the rational 0.1.

Real numbers: Rational plus irrational.

A subset is dense of the rational numbers if all numbers can be approximated to rational numbers, and thus, rational and irrational numbers are dense.

Therefore, option C is the correct answer.

To learn more about the rational and irrational visit:

https://brainly.com/question/15267867.

#SPJ6

Alan's go kart travels 1750 feet per minute and Barry's go kart travels 21 miles per hour whose go kart travels faster round your answer to the nearest tenth

Answers

Answer:

Barry's kart is faster as he goes 21 Mph while Alan only goes 19.89 Mph.

Step-by-step explanation:

First to get this answer we need to convert feet per minute to miles per hour. So we multiple 1750 feet per minute by 60 to get a hour. This gets up 105000 total feet in a hour. Now we need to divide this by 5280, which is how long a mile is, to get 19.89 miles which is how many miles he travels in a hour making him travel 19.89 mph which is less than 21 making Barry faster.

Seismosaurus is the longest known dinosaur. It
measured 1800 inches. How far would 3000
Seismosaurus dinosaurs span if they were
placed head to tail? Write your answer in
scientific notation.

Answers

Answer:

5.4 * 10^6 in.

Step-by-step explanation:

3000 * 1800 in. = 5,400,000 in. = 5.4 * 10^6 in.

5. Which of the following describe the transformation rule (x, y) – (x + 1, - 2)
Move left two, down one
Move right one, down two
Move right two, down one
Move left two, up one

Answers

Answer:

shift to the right 1 unit, shift down 2 units

Step-by-step explanation:

because x+1 which mean shift to the right 1 unit as +1 is positive

down 2 because y is -2 as well as shift down 2 units

Other Questions
what is n to the -2 power times n to the 6 power Why was Babylon considered the culture center? On a pleasant summer day, you are sitting at a sidewalk coffee shop drinking an iced coffee and reading a book when you see a man walking toward you wearing a parka, snow pants, heavy winter boots, and goggles. You ask why he's dressed that way, and he says that a friend of his wants to sell his winter camping gear, and that he just wants to try it out before buying it is heading back to his friend's place now. Which answer best describes his behavior? Help please!! Thank you - 17 RATIONAL OR IRRATIONA During a backgammon game, Jimmy rolled two dice on the board. Find the probability that the sum obtained was greater than 8 given that the number on one die was a 6 Solve for x.X^2-6x+10=2 Describe the function of each organelle.Nucleus 1. ...gas is the major contributors of global warming. 2. Which is the most abundant greenhouse gas.? 3. Which gas is largely responsible for the greenhouse effect? What characteristics do economists use to categorize goods into groups? 2^5*2^3/4^4 plz answer Rhonda has one sheet of paper. She cuts it into thirds and stacks the 3 sheets. If she completes this process a total of 5 times, how many sheets thick will the resulting stack be? by the way sh only has to complete the process 27 before it reaches the moon. When you first see something, the cornea projects an image onto your:A. optic nerveB. occipital lobeO c. retinaD. iris The pilot landed the plane with such ease. What is the action verb and linking verb in the sentence I NEED HELP ASAP!7x-3y= 6; = 3 What was one reason Congress gave for wanting to remove President Johnson from office? A. Johnson was violating the Civil Rights Act of 1866. B. Johnson was abusing his presidential powers. C. Johnson had failed to provide his own Reconstruction plan. D. Johnson had created new Cabinet positions without approval. This question is confusing. Please help me! How do I solve this problem? If Maggies difficulties are viewed as behaviors seen in other children her age, but as more frequent, intense, and detrimental to her development, then her behaviors are seen as dimensional.True / False. PLZ HELP as a result of the change in the migratory pattern, the two groups of blackcaps(those that migrate to England vs those that migrate to spain) have different mating seasons. Blackcaps that migrate to England have an earlier mating season. Scientists believe that this difference may result to two different species of birds. This shows: a) the effects of symbiosis b)the effects of local evolution or migration on an ecosystem c) the difference between biotic factors in an ecosystem d) how migratory patterns change in response to alterations in an ecosystem